Suppose y varies directly with x. If y is 8 when x is 12, find y when x is 18.​

Answers

Answer 1

[tex]\large\textsf{\textbf{Answer\::}}[/tex]

Please consult the "calculations" section of this ans .

[tex]\large\textsf{\textbf{Calculations\::}}[/tex]

If y varies directly with x , the equation looks like this :

y=kx , where k is the constant of proportionality

Plug in the values :

8=k/12

Multiply times 12 on both sides :

96=k

So the constant of proportionality is 96 .

Now solve for y when x is 18 as follows :

y=96/18

y≈5.33

[tex]\footnotesize\texttt{hope\:helpful~}[/tex]


Related Questions

Find the value of x in each triangle. . X= 60° 60° x° ​

Answers

Answer:

60 degrees

Step-by-step explanation:

Triangle = 180 degrees

60+60+X=180

120+X=180

X=60

F(x, y, z) = yzi 9xzj exyk, c is the circle x2 y2 = 9, z = 1

Answers

The value of curlF.dS is 72π if the F(x, y, z) = yzi 9xzj exyk, c is the circle x2 y2 = 9, z = 1

What is integration?

It is defined as the mathematical calculation by which we can sum up all the smaller parts into a unit.

We have:

[tex]\rm F(x, y, z) = yzi+ 9xzj+ e^{xy}k[/tex]

And curl is x²+y²= 9 and z =1

In the parametric form:

[tex]\rm \vec{r}(t) = 3cost \vec{i}+3sint \vec{j}+\vec{k}[/tex]  0 ≤ t ≤ 2π

First two component represent the circle and last one represent the z =1

Using Stoke's theorem:

[tex]\rm \int\limits\int\limits_S {curl \ \ve{F}.d\vec{S}} = \int\limits_C {\vec{F}} \, .d\vec{r } = \int\limits^{2\pi}_0 {\fec{F}(\vec{r}(t)).\vec(r)t} \, dt[/tex]

Here:

[tex]\rm \vec{F}(\vec{r}(t)) = 3sint \vec{i}+27cost \vec{j}+e^{cost.sint}\vec{k}[/tex]

Now calculate the dot product of curl F and dS we get:

[tex]\rm \int\limits\int\limits_S {curl \ \ve{F}.d\vec{S}} = \int\limits^{2\pi}_0 (-9sin^2t+81cos^2t)dt[/tex]

After solving the above integral, we will get:

[tex]\rm \int\limits\int\limits_S {curl \ \ve{F}.d\vec{S}} = 72\pi[/tex]

Thus, the value of curlF.dS is 72π if the F(x, y, z) = yzi 9xzj exyk, c is the circle x2 y2 = 9, z = 1

Learn more about integration here:

brainly.com/question/18125359

#SPJ4

Dion flips a coin 20 times and records if it comes up heads. if getting heads is a success, what is the probability of a success on each roll? 0.2 0.3 0.5 1.00

Answers

The probability of a success on each roll of the coin that is filipped by Dion is 0.5.

What is the probability?

Probability is the likelihood that a stated event would occur. The odds the event occurs is 1 and the odds that the event does not happen is 0. If a coin is flipped, there is 50% chance of getting either a head or a tail.

To learn more about probability, please check: https://brainly.com/question/13234031

#SPJ4

Answer:

its c

Step-by-step explanation:

.5 on edge! just took the test

please solve for me. please be right

Answers

Answer:

It would be 60

Step-by-step explanation:

So 3(3*5 + 5) so 3*5 would be 15, 15+5=20, and 20*3=60

This is really simple if you can't figure this out, you might as well go back to 1st grade and below

pls try before asking

The right Answer is 20 not really that hard but yea it’s 20

An employee of a store's gift wrapping center is wrapping 8 gifts
same size box. The dimensions of the box are shown to the right
A Find the volume of the box to see how much space is available to place
packages. Show your work
B. How much wrapping paper is needed to wrap ONE box? Show your
work
If there is only 160 square feet of wrapping paper left, will the employee
be able to wrap all of the gifts? Explain.

Answers

Part A:

The area A of a rectangle is A = bh, where b is the base of the rectangle and h is the height. The area of each rectangle with side lengths 1.5 ft and 2 ft is 1.5 × 2 = 3ft2. Since there are two rectangles with these dimensions, the combined area is 2 × 3 = 6 ft2. The area of each rectangle with side lengths 1.5 ft and 2.5 ft is 1.5 × 2.5 = 3.75 ft2. The area of each rectangle with side lengths 2 ft and 2.5 ft is 2 × 2.5 = 5 ft2. Since there are two rectangles of each type, the combined area is 2 × 3.75 + 2 × 5 =17.5 ft2. So, the total surface area of the box is 6 ft2+ 17.5 ft2 = 23.5 ft2

Part B:

The employee needs to wrap 8 boxes, each with a surface area of 23.5 ft2. So, the combined surface area needing to be wrapped is 8 × 23.5 = 188 ft2. Since there is only 160 square feet of wrapping paper left, the employee will not be able to wrap all of the gifts

Question
One hundred grams of radium are stored in a container. The amount R (in grams) of radium present after t years can be modeled by R=100e−0.00043t. After how many years will only 5 grams of radium be present? Round your answer to the nearest whole year.

It will take about ??
years for only 5 grams of radium to be present.

Answers

The number of years it would take for radium to be 5 grams is 6967 years.

After how many years will only 5 grams of radium be present?

The formula that is used to represent continuous compounding or continuous decay is:

FV = A x [tex]e^{-t}[/tex]  x N

Where:

A= amounte = 2.7182818N = number of yearsr = interest rate

5 = 100 x e^−0.00043 x t.

In order to determine the value of t, take the following steps:

Divide both sides by 100

5/100 = e^−0.00043 x t.

Then take the In of both sides

= 6967 years

To learn more about how to determine the number of years, please check: https://brainly.com/question/25760893

#SPJ1

Which of the following is NOT equivalent to 16?
A:22 × 22
B:42
C:4(22)
D:(4 × 2)2

Answers

Answer:

D.(4x2)^2 = 64 not 16

Can someone help me on this i am stuck on it thanks

Answers

Answer:

36

Step-by-step explanation:

Given:

A kitchen shop sells bowls in sizes small, medium and large.

Draw a Venn diagram with 3 intersecting circles. Label each circle:

S = small bowlsM = medium bowlsL = large bowls

----------------------------------------------------------------------------------------------

Given:

5 people bought all three sizes of bowl.

Enter "5" in the central overlapping section of the Venn diagram.

----------------------------------------------------------------------------------------------

Given:

7 people bought a small bowl and a large bowl.

We know that 5 people bought all 3 bowls, so to find the number who bought a small and large bowl (but not a medium bowl):

⇒ 7 - 5 = 2

Enter "2" in the section S and L but not M.

----------------------------------------------------------------------------------------------

Given:

In total, 27 people bought a small bowl, of which 12 also bought a medium bowl.

First, calculate the number of people who bought only a small and medium bowl (but not a large bowl).  We know that 5 people bought all 3 bowls, so subtract 5 from 12.

Enter "7" in the section S and M but not L

A total of 27 people bought a small bowl, so to find the number who bought a small bowl only (not a medium and/or large bowl), subtract the other found numbers from 27:

⇒ 27 - 5 - 2- 7 = 13

Enter "13" in the S only section.

----------------------------------------------------------------------------------------------

Given:

In total, 66 people bought a medium bowl, of which 15 also bought a large bowl.

First, calculate the number of people who bought only a large and medium bowl (but not a small bowl).  We know that 5 people bought all 3 bowls, so subtract 5 from 15.

Enter "10" in the section M and L but not S

A total of 66 people bought a medium bowl, so to find the number who bought a medium bowl only, subtract the other found numbers:

⇒ 66 - 5 - 7 - 10 = 44

Enter "44" in the M only section.

----------------------------------------------------------------------------------------------

Given:

On a given day, 100 people each bought at least one bowl from the shop.

To find "L only" subtract all the known numbers from 100:

⇒ 100 - 5 - 2 - 7 - 13 - 10 - 44 = 19

Enter "19" into the L only section.

----------------------------------------------------------------------------------------------

Finally, to work out the total number of people that bought a large bowl, add up the numbers in the L circle:

⇒ 2 + 5 + 10 + 19 = 36

b) Work out the value of (2.92 × 106) + (4 × 10¯²) Give your answer in standard form.​

Answers

Step-by-step explanation:

[tex]3.0956 \times {10}^{2} [/tex]

is correct answer

square root of 3 diveded by the square root of 2

Answers

Answer:

1.2 or if you round it is 1

Answer:

1.224744871 or (square root of 6)/2

Step-by-step explanation:

the price of an item has been reduced by 20%. the origianal price was 76

Answers

Answer:

$60.8

Step-by-step explanation:

Twenty percent of 76 is,

→ 20% of 76

→ (20/100) × 76

→ 1520/100

→ [ $15.2 ]

The reduced price of the item is,

→ 76 - 15.2

→ [ $60.8 ]

Hence, the price is $60.8.

could i get help pleaseeeee

Answers

aboustouly tell me ur problem

Which expression is equivalent to 5 sqrt 32x5y10z15?

Answers

The expression equivalent to 5 sqrt 32x5y10z15 is 2xy2z3

If the right triangle's dimensions are enlarged by 3 units, the new height would be *blank* units. Just write the numerical answer.

Answers

Answer

5

Step-by-step explanation:

there are already 2 units in height therefore 2 plus 3 is 5

What is the sum of 7 1/12 + 8 1/12 + 3 1/12?

Answers

Answer:

8,064 I'm sure that's is answer

Solve this proportion: ​

Answers

The answer to this proportion would be X=4

Answer:

4

Step-by-step explanation:

14 is 2/3 of 21

6 x 2/3

4

help me please please

Answers

13 in 14 in 16 21 the cyclist

Use the given values of n and p to find the minimum usual value μ- 20 and the maximum usual value μ+20. Round your answer to the nearest hundredth unless otherwise noted.
n=713, p= 4/5

Answers

Answer:

μ - 2σ = 549.04μ + 2σ = 591.76

Step-by-step explanation:

Given the parameters of binomial distribution

n = 713, p = 4/5

We need to find the mean μ and the standard deviation σ to calculate required values

Follow the steps below

1. Find the mean

μ = n·p = 713*(4/5) = 570.4

2. Find the variance

σ² = np(1 - p) = 713*(4/5)*(1 - 4/5) = 114.08

3. Find the standard deviation

σ = √114.08 = 10.68 (rounded)

Find the minimum usual value

μ - 2σ = 570.4 - 2*10.68 = 549.04

Find the maximum usual value

μ + 2σ = 570.4 + 2*10.68 = 591.76

Answer:

Min:549.04; Max:591.76

Step-by-step explanation:

a 3. A card is pulled from a standard deck of cards.
Can you find P(queen or hearts)? (make sure you aren't double counting!!!)l

Answers

Answer:    4/13

Explanation:

There are 13 hearts, including the queen of hearts. Then there are 3 more queens to get us to 13+3 = 16 cards we're after.

There are 16 cards we want out of 52 total, so the probability is 16/52 = (4*4)/(4*13) = 4/13

Write down a number that has the same value as:
1-13.51

Answers

Answer:

The answer would have to be 5 but im not sure cause there are no answer choices.

Step-by-step explanation:

What’s a 1/3 as a whole number

Answers

Answer:

a repeating term of .3

Step-by-step explanation:

1/3 = 1 divided by 3

1 divided by 3 = .33333333333333333333

that is called a repeating term

[tex]\frac{1}{3}[/tex] = .3

Answer: 1/3 is .3333333 on going

it cant be a whole number

The Simpsons rented a trailer that was 8 feet long and 5 feet wide. If they load the trailer with 1-by-1-by-1-foot boxes to a height of 3 feet, how many boxes can be loaded onto the trailer?​

Answers

The total number of boxes that can be loaded onto the trailer is 27.

Here we have to determine the maximum number of boxes.

That can be stored in a locker by the volume of one box.

From the given information volume of the box is Vb and the volume f the locker is Vl.

What is the volume of the box?

[tex]Volume=Length\times width\times height[/tex]

Therefore we get,

[tex]V_b=4\times3\times2\\V_b=24ft^3[/tex]

The volume of the locker is

[tex]V_L=12\times6\times 9\\V_L=684ft^3[/tex]

Therefore the number of the maximum boxes are

[tex]\frac{VL}{V_b} =\frac{648}{24} \\=27[/tex]

Therefore, The total number of boxes can be loaded onto the trailer are 27.

To learn more about the volume visit:

https://brainly.com/question/1972490

#SPJ1

4 + 7 + 9 + 5 - 44? pls HELPPPPP!!!!!!!!!!!!!!!

Answers

Answer:

-19.

Step-by-step explanation:

4 + 7 + 9 + 5 - 44

Doing the adds first;

= 25 - 44

= -19.

Answer: -19

Step-by-Step Explanation:

=> 4 + 7 + 9 + 5 - 44
= 11 + 9 + 5 - 44
= 20 + 5 - 44
= 25 - 44
=> -19

Find the probability of each event
A fair coin is flipped ten times. What is
the probability of the coin landing tails up
at least nine times?

Answers

Answer:

11/1024.

Step-by-step explanation:

Binomial Probability distribution.

This is the probability of 9 tails or 10 tails being flipped.

Prob ( 10 tails) = (1/2)^10 = 1/1024

Prob ( 9 tails) = 10C9 * 1/2^9* 1/2 = 5/512

Required probability =  1/1024 + 5/512

= 11/1024.

which has the same ratio as 8 inches/1 foot?

Answers

Answer:

16 inches/2 feet

Step-by-step explanation:

We already have our ratio of 8:1, so we can multiply this ratio by any number of out choosing and the ratio will still be the same.

What is the area of a square that has a length of x-5?

Answers

Answer:

x^2 - 10x + 25

Step-by-step explanation:

(x-5) (x-5)

= x^2-10x+25

Answer:

Step-by-step explanation:

[tex]\text{Formula for the area of a square:}[/tex]

[tex]A=s^2[/tex] [tex](\text{"S" represents the side length} )[/tex]

[tex]\text{Given}[/tex]:

[tex]\text{The side length is x - 5, so we can plug that into the equation}[/tex]

[tex]\text{Solving}[/tex]:

[tex]A=s^2[/tex][tex]A=(x-5)^2[/tex][tex]A=x^2-10x+25[/tex]

State the degree and dominant term of this polynomial
function. (2 marks)[tex]f(x)=2x(x-3)^3(x+1)(4x-2)[/tex]

Answers

The degree of the polynomial f(x) = 2x(x - 3)³(x + 1)(4x - 2) is 6, and the dominant term is - 216x²

The degree of the polynomial?

The polynomial function is given as:

f(x) = 2x(x - 3)³(x + 1)(4x - 2)

To determine the degree, we simply add the multiplicities.

So, we have:

Degree = 1 + 3 + 1 + 1

Evaluate

Degree = 6

Hence, the degree of the polynomial is 6

The dominant term of the polynomial

We have:

f(x) = 2x(x - 3)³(x + 1)(4x - 2)

Expand

f(x) = 8x⁶ - 68x⁵ + 176x⁴ - 72x³ - 216x² + 108x

The term with the highest absolute value is - 216x²

Hence, the dominant term is - 216x²

Read more about polynomials at:

https://brainly.com/question/4142886

#SPJ1

What kind of shape is this ?

Answers

Answer:

prism

Step-by-step explanation:

when all the sides are joined, it will reveal the shape to become a prism.

Prism because it is a three dimensional shape


David needs to choose a tie for a wedding. He has 4 solid color ties, 5
striped ties, and 3 cartoon ties. If his wife picks a tie for him at random,
what of the following is the probability that she will NOT choose a solid
color tie?
elect one:

Answers

The probability of not choosing the solid color tie by David's Wife will

be [tex]\frac{2}{3}[/tex].

What is Probability?

Probability means possibility. It is a branch of mathematics that deals with the occurrence of a random event.

Number of solid Color ties= 4Number of cartoon ties= 3Number of stripped ties= 5

Total Number of ties= 4+3+5 =12

Probability of not choosing the solid color tie

=[tex]\frac{Total\; ties- \; Number\; of \;Solid\; ties}{Total\; number \;of \;ties}[/tex]

=[tex]\frac{12-4}{12}[/tex]

= [tex]\frac{8}{12}[/tex]

=[tex]\frac{2}{3}[/tex]

Learn more about probability here:

https://brainly.com/question/2150559

#SPJ1

2. Three times some number, decreased by five is no less than 16.

Written as a inequality, what will it be?

Answers

Answer:

Read explanation

Step-by-step explanation:

The equation is written as: [tex]3x-5\geq 16[/tex]

You can make it a 2 step equation.

[tex]3x-5\geq 16\\[/tex]

16+5=21

[tex]3x\geq 21[/tex]

21/3

     - - - - - - - - - - - - - - - - - - - - - - - - - - - - - - - - - - - - - - - - -

[tex]\blue{\textsf{\textbf{\underline{\underline{Question:-}}}}[/tex]

         3 times some number decreased by 5 is no less than 16.

[tex]\blue{\textsf{\textbf{\underline{\underline{Answer:-}}}}[/tex]

3n-5≥16

[tex]\blue{\textsf{\textbf{\underline{\underline{How\:to\:Solve:-}}}}[/tex]

              First, notice it says "no less than 16"

So the expression can't be less than 16, so it's greater than or equal to 16.

Now, let the number be n.

Multiply 3 times n:

3n

"Decreased by 5" means you subtract 5:-

3n-5

Inequality:- 3n-5≥15

Solve:

3n-5≥15

3n≥15+5

3n≥20

n≥20/3

Good luck.

              - - - - - - - - - - - - - - - - - - - - - - - - - - - - - - - - - - - - - - - - - - - - - -

Other Questions
PLEASE HELP ASAP!Confucius developed his teachings because he hoped to: A. give citizens the power to choose their own leaders.B. help young people question their parents and teachers.C. stop people from worshipping false gods. D. bring stability to his home during a period of chaos. If I could keys u I would, loov u guys Which operation should be completed first to find the value of the expressionbelow?12 + 30 = (6 - 3) +112 + 3030 : 66 - 33+1 A coating machine costs $125,000. The coating solution costs $.15 per piece. You expect the coating machine to coat 50,000 parts over its useful life. What is the machine cost per part? A cats pulse was measured at 120 beats per minute. How many beats did the veterinarian count during the 15 second measuring interval? What percent of 72 is27? Expression 3x^2-105x-60 and (5x-20)(6x+3)Which expression make it easier to find the values of x that make the equation f(x)= 0 true? Explainfind the values of x that make f(x) =0 HELP ME PLEASE SOMEONE! - A grid shows the positions of a subway stop and your house. The subway stop is located at (-36), and your house is located at (-7, 2). What is the distance, to the nearest unit, between yourhouse and the subway stop?0609O 11O 13 What is grit? How has grit helped Patsy Mink pursue her goals in life? Damien is being relocated to a new city abroad for work, and he is going to need a longer-term hotel stay while he acclimates to the new locale and culture. He wants a space that feels like home, but he also wants access to a grocery shopping service and housekeeping since his time will be focused on his new role. Which Marriott property is most likely to meet his needs? 2. JAVON MAILS 3 PACKAGES. THE FIRST WEIGHS 1.428 POUNDS, THESECOND WEIGHS 12, 35 POUNDS, AND THE THIRD WEIGHS 18.3 POUNDS.WHAT IS THE TOTAL WEIGHT OF THE PACKAGES? Find the value of x Thank you anyone helping Which What-If Analysis tool would you use to solve the following problem? You want to determine how your company can stay profitable by varying the commission rates for your 12 sales people. Last year, a professional basketball teamwon 60 games. This year the team won 9fewer games than it won last year. Whatis the percent decrease in the number ofgames the team won? (7.4D Lesson 2) What is the empirical formula of a compound containing 90 g carbon 11 g hydrogen and 35 g nitrogen How much energy does a 445 nm wave of light have? (The speed of light in avacuum is 3.00 x 108 m/s, and Planck's constant is 6.626 x 10-34 Js.) What is cell division Can someone write me a horror story about rain? ( 3 paragraphs ).( brainliest and 50 points ) help me solve this problem please